LSAT 33 – Section 3 – Question 14

You need a full course to see this video. Enroll now and get started in less than a minute.

Target time: 1:00

This is question data from the 7Sage LSAT Scorer. You can score your LSATs, track your results, and analyze your performance with pretty charts and vital statistics - all with a Free Account ← sign up in less than 10 seconds

Question
QuickView
Type Tags Answer
Choices
Curve Question
Difficulty
Psg/Game/S
Difficulty
Explanation
PT33 S3 Q14
+LR
Argument part +AP
A
2%
162
B
0%
150
C
8%
162
D
18%
163
E
71%
168
141
154
168
+Harder 148.18 +SubsectionMedium

This is an Argument Part question, so we have to describe the role played in the argument by the statement that "many major cities had similar ratios of police officers to citizens, yet diverged widely in their crime rates."

Like all other LR questions, we try to identify the conclusion and the premises first.

This passage is sneaky. It start by telling us that "many people believe X". Normally, when we encounter a passage that starts like that, what's the upshot? What's the conclusion?

That those people are wrong in their moronic beliefs.

I mean, if I just told you "Look man, I know that lots of people believe X, but Y". You actually get a lot of information out of this right?

You must hear me communicating to you that those people are wrong. The structuring of that sentence, using "but" says that much.

In this passage, we're suppose to hear that increasing the number of police officers is not the only way to remedy crime. That's the unspoken conclusion.

Why should we believe it?

Just look at all these major cities with about the same number of cops to citizens (say, 1 cop for every 1,000 citizens). Yet, they all have very different levels of crime.

We are prodded to draw the conclusion that there must be some other factor that influence the level of crime. In other words, number of cops isn't the only factor. That's answer choice (E).

(C) is wrong because it's not clear what counts as a "proof". If they mean validity, then certainly this argument doesn't meet that high standard. Additionally, (C) says there are other factors that are "more important" than the number of cops. We have some information that there exists other factors, maybe. But we have no information about the relative causal strength of those factors. Which one is more important?

(D) is wrong because the idea of "having no effect" is very different from the idea of "being one causal component amongst many".

Take PrepTest

Review Results

Leave a Reply